Question

In: Economics

Two firms, Firm 1 and Firm 2, compete by simultaneously choosing prices. Both firms sell an...

Two firms, Firm 1 and Firm 2, compete by simultaneously choosing prices. Both firms sell an identical product for which each of 100 consumers has a maximum willingness to pay of $40. Each consumer will buy at most 1 unit, and will buy it from whichever firm charges the lowest price. If both firms set the same price, they share the market equally. Costs are given by ??(??)=16??ci(qi)=16qi. Because of government regulation, firms can only choose prices which are integer numbers, and they cannot price above $40.

Answer the following:

a) (0.25 point) If Firm 1 chooses ?1=33p1=33, Firm 2's best response is to set what price?

b) (0.25 point) If Firm 2 chooses the price determined in the previous question, Firm 1's best response is to choose what price?

c) (1 point) If Firm 1 chooses ?1=7p1=7, Firm 2's best response is a range of prices. What is the lowest price in this range?

d) (1 point) Now suppose both firms are capacity-constrained: Firm 1 can produce at most 26 units, and Firm 2 can produce at most 45 units. If firms set different prices, consumers will first buy from the firm charging the lower price. Once that firm's supply is exhausted, consumers will buy from the firm charging the higher price until that firm's supply is exhausted. What is Firm 1's equilibrium profit?

Solutions

Expert Solution

Answer :

Solution (a) :-

If firm 1 choose P1 = 33, firm 2 can choose next lower integer i.e P2 = 32. Since the customers will move to the firm charging lowest price. The best response of firm 2 will be to set price just the next lower integer. Therefore, P2= 32.

Solution (b) :-

If firm 2 chooses to set the price at P2= 32. It will be in the best interest of firm 1 to set price at the next lower integer i.e P1= 31. Since the customers will move to the firm charging lowest price. Its beneficial to charge just a little bit less (next integer) than the other firm.

Solution (c) :-

If firm 1 chooses price p1 = 7. Firm 2 can choose any price between 7 to 16. The lowest price in this will be 7. But we that firms choose price equal to Marginal cost.

Thus, the lowest price in this range is $7.

In this case c(1)= 16q1

MC(1) =16

MC= P= 16 at opimal level.

So a rational producer will choose p =16(maximum charge price)

Solution (d) :-

Firm 1 will produce 26 units and firm 2 will produce 45 units. so a total of 26+45= 71 customers will be served out of 100.

Firm 1 can charge $40 that is the maximum willingness to pay of the customers. Since after 26 units customers will move to firm 2.

Profit of firm 1 = TR-TC

= 40×26 - 16×26

= 1040 - 416

Profit of Firm 1 = 624


Related Solutions

Consider two firms that sell differentiated products and compete by choosing prices. Their demand functions are...
Consider two firms that sell differentiated products and compete by choosing prices. Their demand functions are Q1 = 72 – 3P1 + 2P2 and Q2 = 72 – 3P2 + 2P1 where P1 and P2 are the prices charged by firm 1 and 2, respectively, and Q1 and Q2 are the corresponding demands. All production costs are assumed to be zero. (a) Suppose the two firms set their prices simultaneously and non-cooperatively. Find the resulting Bertrand-Nash equilibrium. What price does...
problem 1. Suppose two firms facing a demand D(p) compete by setting prices simultaneously (Bertrand Competition)....
problem 1. Suppose two firms facing a demand D(p) compete by setting prices simultaneously (Bertrand Competition). Firm 1 has a constant marginal cost c1 and Firm 2 has a marginal cost c2. Assume c1 < c2, i.e., Firm 1 is more efficient. Show that (unlike the case with identical costs) p1 = c1 and p2 = c2 is not a Bertrand equilibrium. how do you solve this?
2. Suppose an industry consists of two firms that compete in prices. Each firm produces one...
2. Suppose an industry consists of two firms that compete in prices. Each firm produces one product. The demand for each product is as follows: q1 = 25 − 5p1 + 2p2 q2 = 25 − 5p2 + 2p1 The cost functions are C(qi) = 2 + qi for i = 1, 2. The Nash equilibrium price is 3.75 (e) What is the percentage markup of price over marginal cost here (this is called the Lerner index)? Do the firms...
QUESTION 1 Two firms compete by choosing price. Their demand functions are Q1 = 20 -...
QUESTION 1 Two firms compete by choosing price. Their demand functions are Q1 = 20 - P1 +P2 and Q2 = 20 - P2 +P1 where P1 and P2 are the prices charged by each firm, respectively, and Q1 and Q2 are the resulting demands. Note that the demand for each good depends only on the difference in prices; if the two firms colluded and set the same price, they could make that price as high as they wanted, and...
If firms compete by choosing quantities of output, is there an advantage to moving later or...
If firms compete by choosing quantities of output, is there an advantage to moving later or sooner? How might a firm commit to go sooner?
Two firms, a and b, compete in a market to sell homogeneous products with inverse demand...
Two firms, a and b, compete in a market to sell homogeneous products with inverse demand function P = 400 – 2Q where Q = Qa + Qb. Firm a has the cost function Ca = 100 + 15Qa and firm b has the cost function Cb = 100 + 15Qb. Use this information to compare the output levels, price and profits in settings characterized by the following markets: Cournot Stackelberg Bertrand Collusion
Consider two firms, Firm A and Firm B, who compete as duopolists. Each firm produces an...
Consider two firms, Firm A and Firm B, who compete as duopolists. Each firm produces an identical product. The total inverse demand curve for the industry is P=250-QA+QB. Firm A has a total cost curve CAQA=100+QA2. Firm B has a total cost curve CBQB=100+2QB. Suppose for now, only Firm A exists (QB=0). What is the Monopoly equilibrium quantity and price? What is Firm A’s profit? Find the Nash Cournot equilibrium price and output level. What are the firms’ profits? Find...
Suppose there are two firms in a market who each simultaneously choose a quantity. Firm 1’s...
Suppose there are two firms in a market who each simultaneously choose a quantity. Firm 1’s quantity is q1, and firm 2’s quantity is q2. Therefore the market quantity is Q = q1 + q2. The market demand curve is given by P = 100 – 2Q. Also, each firm has constant marginal cost equal to 10. There are no fixed costs. The marginal revenue of the two firms are given by: MR1 = 100 – 4q1 – 2q2 MR2...
Suppose there are two firms in a market who each simultaneously choose a quantity. Firm 1’s...
Suppose there are two firms in a market who each simultaneously choose a quantity. Firm 1’s quantity is q1, and firm 2’s quantity is q2. Therefore the market quantity is Q = q1 + q2. The market demand curve is given by P = 225 - 3Q. Also, each firm has constant marginal cost equal to 9. There are no fixed costs. The marginal revenue of the two firms are given by: MR1 = 225 – 6q1 – 3q2 MR2...
Suppose there are two firms in a market who each simultaneously choose a quantity. Firm 1’s...
Suppose there are two firms in a market who each simultaneously choose a quantity. Firm 1’s quantity is q1, and firm 2’s quantity is q2. Therefore the market quantity is Q = q1 + q2. The market demand curve is given by P = 160 - 2Q. Also, each firm has constant marginal cost equal to 10. There are no fixed costs. The marginal revenue of the two firms are given by: MR1 = 160 – 4q1 – 2q2 MR2...
ADVERTISEMENT
ADVERTISEMENT
ADVERTISEMENT